Solvable group $G$ isn't a cyclic $p$-group











up vote
-1
down vote

favorite
1












Let $G$ be a nontrivial finite solvable group. We want to show that $G$ has proper subgroups like $A$ and $B$ such that $G=AB$ iff $G$ isn't a cyclic $p$-group.



I use the below fact to solve the problem.



" Let $G$ be a nontrivial finite group. $G$ has one and only one maximal subgroup iff $G$ be a cyclic group of order $p^{m}$ such that $p$ is a prime number. "



I can solve one side but I have problem on the other side.



I suppose that $G$ isn't a cyclic $p$-group. Therefore $G$ has more than one maximal subgroup. Let $M$ and $N$ be maximal subgroups of $G$ such that $M neq N$. I want to show $G = MN $ but I don't know how.










share|cite|improve this question




















  • 1




    $MN$ is a subgroup of $G$ and contains both $M$ and $N$. Does that help?
    – user3482749
    Nov 15 at 16:21










  • @user3482749Why $MN$ is a subgroup?If so, yes, we have $ M lneqq MN leq G $ and as $M$ is a maximal subgroup of $G$, we have $G=MN$.
    – Yasmin
    Nov 15 at 16:43












  • The result is false when $G = C_{p^infty}$, which is an infinite abelian group in which all proper subgroups are finite. And your fact is wrong when $G = C_{p^infty} times C_p$, which has the unique maximal subgroup $C_{p^inf ty}$.
    – Derek Holt
    Nov 15 at 21:13








  • 1




    Your original version said nothing about finiteness, so why would you expect anyone to think that you were talking about finite groups?
    – Derek Holt
    Nov 16 at 8:05






  • 1




    Anyway, you can solve your problem by taking $M$ to be a normal subgroup of prime index, and then $MN$ is a subgroup.
    – Derek Holt
    Nov 16 at 8:05















up vote
-1
down vote

favorite
1












Let $G$ be a nontrivial finite solvable group. We want to show that $G$ has proper subgroups like $A$ and $B$ such that $G=AB$ iff $G$ isn't a cyclic $p$-group.



I use the below fact to solve the problem.



" Let $G$ be a nontrivial finite group. $G$ has one and only one maximal subgroup iff $G$ be a cyclic group of order $p^{m}$ such that $p$ is a prime number. "



I can solve one side but I have problem on the other side.



I suppose that $G$ isn't a cyclic $p$-group. Therefore $G$ has more than one maximal subgroup. Let $M$ and $N$ be maximal subgroups of $G$ such that $M neq N$. I want to show $G = MN $ but I don't know how.










share|cite|improve this question




















  • 1




    $MN$ is a subgroup of $G$ and contains both $M$ and $N$. Does that help?
    – user3482749
    Nov 15 at 16:21










  • @user3482749Why $MN$ is a subgroup?If so, yes, we have $ M lneqq MN leq G $ and as $M$ is a maximal subgroup of $G$, we have $G=MN$.
    – Yasmin
    Nov 15 at 16:43












  • The result is false when $G = C_{p^infty}$, which is an infinite abelian group in which all proper subgroups are finite. And your fact is wrong when $G = C_{p^infty} times C_p$, which has the unique maximal subgroup $C_{p^inf ty}$.
    – Derek Holt
    Nov 15 at 21:13








  • 1




    Your original version said nothing about finiteness, so why would you expect anyone to think that you were talking about finite groups?
    – Derek Holt
    Nov 16 at 8:05






  • 1




    Anyway, you can solve your problem by taking $M$ to be a normal subgroup of prime index, and then $MN$ is a subgroup.
    – Derek Holt
    Nov 16 at 8:05













up vote
-1
down vote

favorite
1









up vote
-1
down vote

favorite
1






1





Let $G$ be a nontrivial finite solvable group. We want to show that $G$ has proper subgroups like $A$ and $B$ such that $G=AB$ iff $G$ isn't a cyclic $p$-group.



I use the below fact to solve the problem.



" Let $G$ be a nontrivial finite group. $G$ has one and only one maximal subgroup iff $G$ be a cyclic group of order $p^{m}$ such that $p$ is a prime number. "



I can solve one side but I have problem on the other side.



I suppose that $G$ isn't a cyclic $p$-group. Therefore $G$ has more than one maximal subgroup. Let $M$ and $N$ be maximal subgroups of $G$ such that $M neq N$. I want to show $G = MN $ but I don't know how.










share|cite|improve this question















Let $G$ be a nontrivial finite solvable group. We want to show that $G$ has proper subgroups like $A$ and $B$ such that $G=AB$ iff $G$ isn't a cyclic $p$-group.



I use the below fact to solve the problem.



" Let $G$ be a nontrivial finite group. $G$ has one and only one maximal subgroup iff $G$ be a cyclic group of order $p^{m}$ such that $p$ is a prime number. "



I can solve one side but I have problem on the other side.



I suppose that $G$ isn't a cyclic $p$-group. Therefore $G$ has more than one maximal subgroup. Let $M$ and $N$ be maximal subgroups of $G$ such that $M neq N$. I want to show $G = MN $ but I don't know how.







group-theory finite-groups p-groups solvable-groups






share|cite|improve this question















share|cite|improve this question













share|cite|improve this question




share|cite|improve this question








edited Nov 16 at 6:12

























asked Nov 15 at 16:19









Yasmin

10310




10310








  • 1




    $MN$ is a subgroup of $G$ and contains both $M$ and $N$. Does that help?
    – user3482749
    Nov 15 at 16:21










  • @user3482749Why $MN$ is a subgroup?If so, yes, we have $ M lneqq MN leq G $ and as $M$ is a maximal subgroup of $G$, we have $G=MN$.
    – Yasmin
    Nov 15 at 16:43












  • The result is false when $G = C_{p^infty}$, which is an infinite abelian group in which all proper subgroups are finite. And your fact is wrong when $G = C_{p^infty} times C_p$, which has the unique maximal subgroup $C_{p^inf ty}$.
    – Derek Holt
    Nov 15 at 21:13








  • 1




    Your original version said nothing about finiteness, so why would you expect anyone to think that you were talking about finite groups?
    – Derek Holt
    Nov 16 at 8:05






  • 1




    Anyway, you can solve your problem by taking $M$ to be a normal subgroup of prime index, and then $MN$ is a subgroup.
    – Derek Holt
    Nov 16 at 8:05














  • 1




    $MN$ is a subgroup of $G$ and contains both $M$ and $N$. Does that help?
    – user3482749
    Nov 15 at 16:21










  • @user3482749Why $MN$ is a subgroup?If so, yes, we have $ M lneqq MN leq G $ and as $M$ is a maximal subgroup of $G$, we have $G=MN$.
    – Yasmin
    Nov 15 at 16:43












  • The result is false when $G = C_{p^infty}$, which is an infinite abelian group in which all proper subgroups are finite. And your fact is wrong when $G = C_{p^infty} times C_p$, which has the unique maximal subgroup $C_{p^inf ty}$.
    – Derek Holt
    Nov 15 at 21:13








  • 1




    Your original version said nothing about finiteness, so why would you expect anyone to think that you were talking about finite groups?
    – Derek Holt
    Nov 16 at 8:05






  • 1




    Anyway, you can solve your problem by taking $M$ to be a normal subgroup of prime index, and then $MN$ is a subgroup.
    – Derek Holt
    Nov 16 at 8:05








1




1




$MN$ is a subgroup of $G$ and contains both $M$ and $N$. Does that help?
– user3482749
Nov 15 at 16:21




$MN$ is a subgroup of $G$ and contains both $M$ and $N$. Does that help?
– user3482749
Nov 15 at 16:21












@user3482749Why $MN$ is a subgroup?If so, yes, we have $ M lneqq MN leq G $ and as $M$ is a maximal subgroup of $G$, we have $G=MN$.
– Yasmin
Nov 15 at 16:43






@user3482749Why $MN$ is a subgroup?If so, yes, we have $ M lneqq MN leq G $ and as $M$ is a maximal subgroup of $G$, we have $G=MN$.
– Yasmin
Nov 15 at 16:43














The result is false when $G = C_{p^infty}$, which is an infinite abelian group in which all proper subgroups are finite. And your fact is wrong when $G = C_{p^infty} times C_p$, which has the unique maximal subgroup $C_{p^inf ty}$.
– Derek Holt
Nov 15 at 21:13






The result is false when $G = C_{p^infty}$, which is an infinite abelian group in which all proper subgroups are finite. And your fact is wrong when $G = C_{p^infty} times C_p$, which has the unique maximal subgroup $C_{p^inf ty}$.
– Derek Holt
Nov 15 at 21:13






1




1




Your original version said nothing about finiteness, so why would you expect anyone to think that you were talking about finite groups?
– Derek Holt
Nov 16 at 8:05




Your original version said nothing about finiteness, so why would you expect anyone to think that you were talking about finite groups?
– Derek Holt
Nov 16 at 8:05




1




1




Anyway, you can solve your problem by taking $M$ to be a normal subgroup of prime index, and then $MN$ is a subgroup.
– Derek Holt
Nov 16 at 8:05




Anyway, you can solve your problem by taking $M$ to be a normal subgroup of prime index, and then $MN$ is a subgroup.
– Derek Holt
Nov 16 at 8:05















active

oldest

votes











Your Answer





StackExchange.ifUsing("editor", function () {
return StackExchange.using("mathjaxEditing", function () {
StackExchange.MarkdownEditor.creationCallbacks.add(function (editor, postfix) {
StackExchange.mathjaxEditing.prepareWmdForMathJax(editor, postfix, [["$", "$"], ["\\(","\\)"]]);
});
});
}, "mathjax-editing");

StackExchange.ready(function() {
var channelOptions = {
tags: "".split(" "),
id: "69"
};
initTagRenderer("".split(" "), "".split(" "), channelOptions);

StackExchange.using("externalEditor", function() {
// Have to fire editor after snippets, if snippets enabled
if (StackExchange.settings.snippets.snippetsEnabled) {
StackExchange.using("snippets", function() {
createEditor();
});
}
else {
createEditor();
}
});

function createEditor() {
StackExchange.prepareEditor({
heartbeatType: 'answer',
convertImagesToLinks: true,
noModals: true,
showLowRepImageUploadWarning: true,
reputationToPostImages: 10,
bindNavPrevention: true,
postfix: "",
imageUploader: {
brandingHtml: "Powered by u003ca class="icon-imgur-white" href="https://imgur.com/"u003eu003c/au003e",
contentPolicyHtml: "User contributions licensed under u003ca href="https://creativecommons.org/licenses/by-sa/3.0/"u003ecc by-sa 3.0 with attribution requiredu003c/au003e u003ca href="https://stackoverflow.com/legal/content-policy"u003e(content policy)u003c/au003e",
allowUrls: true
},
noCode: true, onDemand: true,
discardSelector: ".discard-answer"
,immediatelyShowMarkdownHelp:true
});


}
});














 

draft saved


draft discarded


















StackExchange.ready(
function () {
StackExchange.openid.initPostLogin('.new-post-login', 'https%3a%2f%2fmath.stackexchange.com%2fquestions%2f2999900%2fsolvable-group-g-isnt-a-cyclic-p-group%23new-answer', 'question_page');
}
);

Post as a guest















Required, but never shown






























active

oldest

votes













active

oldest

votes









active

oldest

votes






active

oldest

votes
















 

draft saved


draft discarded



















































 


draft saved


draft discarded














StackExchange.ready(
function () {
StackExchange.openid.initPostLogin('.new-post-login', 'https%3a%2f%2fmath.stackexchange.com%2fquestions%2f2999900%2fsolvable-group-g-isnt-a-cyclic-p-group%23new-answer', 'question_page');
}
);

Post as a guest















Required, but never shown





















































Required, but never shown














Required, but never shown












Required, but never shown







Required, but never shown

































Required, but never shown














Required, but never shown












Required, but never shown







Required, but never shown







Popular posts from this blog

Plaza Victoria

In PowerPoint, is there a keyboard shortcut for bulleted / numbered list?

How to put 3 figures in Latex with 2 figures side by side and 1 below these side by side images but in...